将方程式正确放入页面

将方程式正确放入页面

我有代码

\documentclass{article}
\usepackage{amsmath}
\usepackage{amssymb}
\DeclareMathOperator{\Tr}{Tr}
\begin{document}

\begin{eqnarray}
\hat K_0(\omega,\kappa_1) &=& \hat J_1(\omega,\kappa_1)J_0(\omega,\kappa_1) \nonumber\\
                         &=& \begin{bmatrix}  r_1^{+}(\omega)e^{i \bar \lambda_1^{-}L} & r_1^{-}(\omega)e^{i \bar \lambda_1^{+}L} \\ r_1^{-}(\omega)e^{i \bar \lambda_1^{-}L} & r_1^{+}(\omega)e^{i \bar \lambda_1^{+}L} \end{bmatrix}
                          \begin{bmatrix} r_0^{+}(\omega) & r_0^{-}(\omega) \\ r_0^{-}(\omega) & r_0^{+}(\omega)\end{bmatrix}  \nonumber\\
                          &=& \begin{bmatrix} r_0^{+}(\omega) r_1^{+}(\omega)e^{i \bar \lambda_1^{-}L} + r_0^{-}(\omega) r_1^{-}(\omega)e^{i \bar \lambda_1^{+}L} & r_0^{-}(\omega) r_1^{+}(\omega)e^{i \bar \lambda_1^{-}L} + r_0^{+}(\omega) r_1^{-}(\omega)e^{i \bar \lambda_1^{+}L} \\ r_0^{+}(\omega) r_1^{-}(\omega)e^{i \bar \lambda_1^{-}L} + r_0^{-}(\omega) r_1^{+}(\omega)e^{i \bar \lambda_1^{+}L} & r_0^{-}(\omega) r_1^{-}(\omega)e^{i \bar \lambda_1^{-}L} + r_0^{+}(\omega) r_1^{+}(\omega)e^{i \bar \lambda_1^{+}L} \end{bmatrix} \nonumber\\
                           &=& \begin{bmatrix} \hat U(\omega) & \hat  G(\omega) \\ \hat V(\omega) & \hat H(\omega) \end{bmatrix}. \nonumber\\
\end{eqnarray}

\end{document}

矩阵方程溢出到页眉。我该如何让它适合我的页面?

答案1

总是使用align..而不是eqnarray已经节省了一半所需空间,那么也许:

在此处输入图片描述

\documentclass{article}
\usepackage{amsmath}
\usepackage{amssymb}
\DeclareMathOperator{\Tr}{Tr}
\begin{document}

\begin{equation}
\begin{aligned}
\hat K_0(\omega,\kappa_1) & = \hat J_1(\omega,\kappa_1)J_0(\omega,\kappa_1)\\
                         &= \begin{bmatrix}  r_1^{+}(\omega)e^{i \bar \lambda_1^{-}L} & r_1^{-}(\omega)e^{i \bar \lambda_1^{+}L} \\r_1^{-}(\omega)e^{i \bar \lambda_1^{-}L} & r_1^{+}(\omega)e^{i \bar \lambda_1^{+}L} \end{bmatrix}
                          \begin{bmatrix} r_0^{+}(\omega) & r_0^{-}(\omega) \\r_0^{-}(\omega) & r_0^{+}(\omega)\end{bmatrix}  \\
                          &= \begin{bmatrix}
\begin{aligned}r_0^{+}(\omega) r_1^{+}(\omega)e^{i \bar \lambda_1^{-}L}\qquad \\{}+ r_0^{-}(\omega) r_1^{-}(\omega)e^{i \bar \lambda_1^{+}L}\end{aligned} &
\begin{aligned}r_0^{-}(\omega) r_1^{+}(\omega)e^{i \bar \lambda_1^{-}L}\qquad \\{}+ r_0^{+}(\omega) r_1^{-}(\omega)e^{i \bar \lambda_1^{+}L}\end{aligned} \\[15pt]
\begin{aligned}r_0^{+}(\omega) r_1^{-}(\omega)e^{i \bar \lambda_1^{-}L}\qquad \\{}+ r_0^{-}(\omega) r_1^{+}(\omega)e^{i \bar \lambda_1^{+}L}\end{aligned} &
\begin{aligned}r_0^{-}(\omega) r_1^{-}(\omega)e^{i \bar \lambda_1^{-}L}\qquad \\{}+ r_0^{+}(\omega) r_1^{+}(\omega)e^{i \bar \lambda_1^{+}L}\end{aligned} \end{bmatrix}\\
                           &= \begin{bmatrix} \hat U(\omega) & \hat  G(\omega) \\ \hat V(\omega) & \hat H(\omega) \end{bmatrix}.\\
\end{aligned}
\end{equation}

\end{document}

答案2

在此处输入图片描述

(红线表示文本边框)

使用mathools(用于\MoveEqLeft)和nccmath(用于中等尺寸mmatrix):

\documentclass{article}
\usepackage{nccmath, mathtools, amssymb}
\DeclareMathOperator{\Tr}{Tr}

%---------------- show page layout. don't use in a real document!
\usepackage{showframe}
\renewcommand\ShowFrameLinethickness{0.15pt}
\renewcommand*\ShowFrameColor{\color{red}}
%---------------------------------------------------------------%

\begin{document}

\begin{align*}
    \MoveEqLeft
\hat K_0(\omega,\kappa_1)                   \notag  \\
    & = \hat J_1(\omega,\kappa_1)J_0(\omega,\kappa_1)
                                            \notag  \\
    & = \begin{bmatrix}
        r_1^{+}(\omega)e^{i \bar \lambda_1^{-}L}
            & r_1^{-}(\omega)e^{i \bar \lambda_1^{+}L} \\
        r_1^{-}(\omega)e^{i \bar \lambda_1^{-}L}
            & r_1^{+}(\omega)e^{i \bar \lambda_1^{+}L}
        \end{bmatrix}
        \begin{bmatrix}
        r_0^{+}(\omega) & r_0^{-}(\omega) \\
        r_0^{-}(\omega) & r_0^{+}(\omega)
    \end{bmatrix}                                   \\
    & = \left[\begin{mmatrix}
        r_0^{+}(\omega) r_1^{+}(\omega)e^{i \bar \lambda_1^{-}L} + r_0^{-}(\omega) r_1^{-}(\omega)e^{i \bar \lambda_1^{+}L} & r_0^{-}(\omega) r_1^{+}(\omega)e^{i \bar \lambda_1^{-}L} + r_0^{+}(\omega) r_1^{-}(\omega)e^{i \bar \lambda_1^{+}L}    \\ r_0^{+}(\omega) r_1^{-}(\omega)e^{i \bar \lambda_1^{-}L} + r_0^{-}(\omega) r_1^{+}(\omega)e^{i \bar \lambda_1^{+}L} & r_0^{-}(\omega) r_1^{-}(\omega)e^{i \bar \lambda_1^{-}L} + r_0^{+}(\omega) r_1^{+}(\omega)e^{i \bar \lambda_1^{+}L} \end{mmatrix}\right]                \notag  \\
    & = \begin{bmatrix}
    \hat U(\omega) & \hat  G(\omega) \\
    \hat V(\omega) & \hat H(\omega)
    \end{bmatrix}.                              \notag
\end{align}
\end{document}

eqnarray注意:不推荐使用,而应使用alignfrom amsmath

相关内容